www.vorhilfe.de
- Förderverein -
Der Förderverein.

Gemeinnütziger Verein zur Finanzierung des Projekts Vorhilfe.de.
Hallo Gast!einloggen | registrieren ]
Startseite · Mitglieder · Impressum
Forenbaum
^ Forenbaum
Status VH e.V.
  Status Vereinsforum

Gezeigt werden alle Foren bis zur Tiefe 2

Navigation
 Startseite...
 Suchen
 Impressum
Das Projekt
Server und Internetanbindung werden durch Spenden finanziert.
Organisiert wird das Projekt von unserem Koordinatorenteam.
Hunderte Mitglieder helfen ehrenamtlich in unseren moderierten Foren.
Anbieter der Seite ist der gemeinnützige Verein "Vorhilfe.de e.V.".
Partnerseiten
Weitere Fächer:

Open Source FunktionenplotterFunkyPlot: Kostenloser und quelloffener Funktionenplotter für Linux und andere Betriebssysteme
Forum "Lineare Algebra - Matrizen" - Basis eines Unterraums
Basis eines Unterraums < Matrizen < Lineare Algebra < Hochschule < Mathe < Vorhilfe
Ansicht: [ geschachtelt ] | ^ Forum "Lineare Algebra - Matrizen"  | ^^ Alle Foren  | ^ Forenbaum  | Materialien

Basis eines Unterraums: Tipps
Status: (Frage) beantwortet Status 
Datum: 15:15 Di 20.03.2012
Autor: Mathegirl

Aufgabe
Folgende Vektoren des [mm] \IR^4 [/mm] sind gegeben:

[mm] v_1=\vektor{1 \\ 2 \\ -1 \\ 0}, v_2=\vektor{1 \\ 0 \\ -1 \\ 0}, v_3=\vektor{0 \\ -1 \\ 3 \\ 2}, v_4=\vektor{0 \\ 1 \\ 0 \\ 1}, v_5=\vektor{2 \\ 2 \\ -1 \\ 1} [/mm]

Bestimme die Basen von [mm] Lin(v_1,v_2,v_3)\cap Lin(v_4,v_5) [/mm] und [mm] Lin(v_1,v_2,v_3)+ Lin(v_4,v_5). [/mm]



Bei dieser Aufgabe habe ich ein Problem, wo ich bisher noch nicht durchgestiegen bin:

Um den Schnitt zu ermitteln gilt gilt ja:

[mm] \lambda_1*\vektor{1 \\ 2 \\ -1 \\ 0}+ \lambda_2* \vektor{1 \\ 0 \\ -1 \\ 0}+\lambda_3*\vektor{0 \\ -1 \\ 3 \\ 2}= \mu_1*\vektor{0 \\ 1 \\ 0 \\ 1}+\mu_2*\vektor{2 \\ 2 \\ -1 \\ 1} [/mm]

Daraus ergibt sich dann folgendes LGS:
[mm] \pmat{ 1 & 1 & 0 & 0 & -2 \\ 2 & 0 & -1 & -1 & -2 \\ -1 & -1 & 3 & 0 & 1 \\ 0 & 0 & 2 & -1 & -1} [/mm]

In Zeilenstufenform gebracht erhält man dann:

[mm] \pmat{ 1 & 0 & 0 & 0 & -1 \\ 0 & 1 & 0 & 0 & -1 \\ 0 & 0 & 3 & 0 & -1 \\ 0 & 0 & 0 & 3 & 1} [/mm]

ich erhalte den freien Parameter [mm] \mu_2=a [/mm] und [mm] \mu_1=-\bruch{a}{3} [/mm]

[mm] V={\mu_1*v_4+\mu_2*v_5/ \mu_2=a , \mu_1=-\bruch{a}{3}, a\in \IR} [/mm] = [mm] {a*\vektor{2 \\ \bruch{5}{3} \\ -1 \\ \bruch{2}{3}}}= {a*\vektor{6 \\ 5 \\ -3 \\2}} [/mm]

also ist [mm] \vektor{6 \\ 5 \\ -3 \\2}eine [/mm] Basis von  [mm] Lin(v_1,v_2,v_3)\cap Lin(v_4,v_5). [/mm]

Könnt ihr mir erklären wie man darauf kommt? Das verstehe ich nicht. Wenn ich nach [mm] \lambda [/mm] und [mm] \mu [/mm] umstelle, dann müsste ich ja 5 Variablen in der Basis haben.


Weiter soll ich eine Basis bestimmen von [mm] Lin(v_1,v_2,v_3)+ Lin(v_4,v_5). [/mm] Aus der Dimensionsformel erhalte ich dim(U+V)=4

kann ich dann eine beliebige Basis des [mm] \IR^4 [/mm] wählen, zum Beispiel die Standardvektoren?

Oder kann ich eine Basis des [mm] \IR^4 [/mm] aus den gegebenen [mm] v_1-v_5 [/mm] wählen? Oder kann ich das LGS in Zeilenstudenform bringen und die nicht Null Zeilen als Basis verwenden?

Hier bin ich mir noch sehr unsicher.


MfG
Mathegirl



        
Bezug
Basis eines Unterraums: Antwort
Status: (Antwort) fertig Status 
Datum: 16:28 Di 20.03.2012
Autor: MathePower

Hallo Mathegirl,



> Folgende Vektoren des [mm]\IR^4[/mm] sind gegeben:
>  
> [mm]v_1=\vektor{1 \\ 2 \\ -1 \\ 0}, v_2=\vektor{1 \\ 0 \\ -1 \\ 0}, v_3=\vektor{0 \\ -1 \\ 3 \\ 2}, v_4=\vektor{0 \\ 1 \\ 0 \\ 1}, v_5=\vektor{2 \\ 2 \\ -1 \\ 1}[/mm]
>  
> Bestimme die Basen von [mm]Lin(v_1,v_2,v_3)\cap Lin(v_4,v_5)[/mm]
> und [mm]Lin(v_1,v_2,v_3)+ Lin(v_4,v_5).[/mm]
>  
>
> Bei dieser Aufgabe habe ich ein Problem, wo ich bisher noch
> nicht durchgestiegen bin:
>  
> Um den Schnitt zu ermitteln gilt gilt ja:
>  
> [mm]\lambda_1*\vektor{1 \\ 2 \\ -1 \\ 0}+ \lambda_2* \vektor{1 \\ 0 \\ -1 \\ 0}+\lambda_3*\vektor{0 \\ -1 \\ 3 \\ 2}= \mu_1*\vektor{0 \\ 1 \\ 0 \\ 1}+\mu_2*\vektor{2 \\ 2 \\ -1 \\ 1}[/mm]
>  
> Daraus ergibt sich dann folgendes LGS:
>  [mm]\pmat{ 1 & 1 & 0 & 0 & -2 \\ 2 & 0 & -1 & -1 & -2 \\ -1 & -1 & 3 & 0 & 1 \\ 0 & 0 & 2 & -1 & -1}[/mm]
>  
> In Zeilenstufenform gebracht erhält man dann:
>
> [mm]\pmat{ 1 & 0 & 0 & 0 & -1 \\ 0 & 1 & 0 & 0 & -1 \\ 0 & 0 & 3 & 0 & -1 \\ 0 & 0 & 0 & 3 & 1}[/mm]
>  
> ich erhalte den freien Parameter [mm]\mu_2=a[/mm] und
> [mm]\mu_1=-\bruch{a}{3}[/mm]
>  
> [mm]V={\mu_1*v_4+\mu_2*v_5/ \mu_2=a , \mu_1=-\bruch{a}{3}, a\in \IR}[/mm]
> = [mm]{a*\vektor{2 \\ \bruch{5}{3} \\ -1 \\ \bruch{2}{3}}}= {a*\vektor{6 \\ 5 \\ -3 \\2}}[/mm]
>  
> also ist [mm]\vektor{6 \\ 5 \\ -3 \\2}eine[/mm] Basis von  
> [mm]Lin(v_1,v_2,v_3)\cap Lin(v_4,v_5).[/mm]
>  
> Könnt ihr mir erklären wie man darauf kommt? Das verstehe
> ich nicht. Wenn ich nach [mm]\lambda[/mm] und [mm]\mu[/mm] umstelle, dann
> müsste ich ja 5 Variablen in der Basis haben.
>  


Es ist richtig,  dass aus obigen Form, die Werte für [mm]\lambda_{1}, \ \lambda_{2}, \ \lambda_{3}, \ \mu_{1}, \ \mu_{2}[/mm] folgen.

Um diesen Vektor, der im Schnitt liegt, zu bestimmen, werden  für
[mm]Lin(v_1,v_2,v_3)[/mm] die ersten 3 Werte benötigt, da
[mm]Lin(v_1,v_2,v_3)[/mm] aus 3 linear unabhängigen Vektoren besteht,
bzw.
[mm]Lin(v_4,v_5)[/mm] die beiden letzten Werte benötigt, da
[mm]Lin(v_4,v_5)[/mm] aus 2 linear unabhängigen Vektoren besteht,


>
> Weiter soll ich eine Basis bestimmen von [mm]Lin(v_1,v_2,v_3)+ Lin(v_4,v_5).[/mm]
> Aus der Dimensionsformel erhalte ich dim(U+V)=4
>  
> kann ich dann eine beliebige Basis des [mm]\IR^4[/mm] wählen, zum
> Beispiel die Standardvektoren?
>  
> Oder kann ich eine Basis des [mm]\IR^4[/mm] aus den gegebenen
> [mm]v_1-v_5[/mm] wählen? Oder kann ich das LGS in Zeilenstudenform
> bringen und die nicht Null Zeilen als Basis verwenden?

>


Du brauchst hier nichts rechnen, da sich der Vektor [mm]v_{5}[/mm] als
Linearkombination von [mm]v_{i}, \ i=1,2,3,4[/mm] ergibt.
Somit kannst Du als Basis [mm]v_{1}, \ v_{2}, \ v_{3}, \ v_{4}[/mm] wählen.

  

> Hier bin ich mir noch sehr unsicher.
>  
>
> MfG
>  Mathegirl
>  


Gruss
MathePower  

Bezug
                
Bezug
Basis eines Unterraums: Frage (beantwortet)
Status: (Frage) beantwortet Status 
Datum: 16:57 Di 20.03.2012
Autor: Mathegirl


> > [mm]\pmat{ 1 & 0 & 0 & 0 & -1 \\ 0 & 1 & 0 & 0 & -1 \\ 0 & 0 & 3 & 0 & -1 \\ 0 & 0 & 0 & 3 & 1}[/mm]
>  
> >  

> > ich erhalte den freien Parameter [mm]\mu_2=a[/mm] und
> > [mm]\mu_1=-\bruch{a}{3}[/mm]
>  >  
> > [mm]V={\mu_1*v_4+\mu_2*v_5/ \mu_2=a , \mu_1=-\bruch{a}{3}, a\in \IR}[/mm]
> > = [mm]{a*\vektor{2 \\ \bruch{5}{3} \\ -1 \\ \bruch{2}{3}}}= {a*\vektor{6 \\ 5 \\ -3 \\2}}[/mm]
>  
> >  

> > also ist [mm]\vektor{6 \\ 5 \\ -3 \\2}eine[/mm] Basis von  
> > [mm]Lin(v_1,v_2,v_3)\cap Lin(v_4,v_5).[/mm]


Ich verstehe nicht wie man darauf kommt! das ist mir nicht klar. Könnt iht mir das nochmal erklären?


MfG
Mathegirl

Bezug
                        
Bezug
Basis eines Unterraums: Mitteilung
Status: (Mitteilung) Reaktion unnötig Status 
Datum: 11:05 Mi 21.03.2012
Autor: Mathegirl

Kann mir nochmal jemand erklären wie ich auf diese Basis komme? Wäre echt nett!


MfG
Mathegirl

Bezug
                        
Bezug
Basis eines Unterraums: Antwort
Status: (Antwort) fertig Status 
Datum: 11:38 Mi 21.03.2012
Autor: angela.h.b.

Hallo,

für $ [mm] v_1=\vektor{1 \\ 2 \\ -1 \\ 0}, v_2=\vektor{1 \\ 0 \\ -1 \\ 0}, v_3=\vektor{0 \\ -1 \\ 3 \\ 2}, v_4=\vektor{0 \\ 1 \\ 0 \\ 1}, v_5=\vektor{2 \\ 2 \\ -1 \\ 1} [/mm] $

suchst Du eine Basis von $ [mm] Lin(v_1,v_2,v_3)\cap Lin(v_4,v_5) [/mm] $.

Du hast festgestellt, daß im Schnitt gerade die Vektoren liegen, für die man [mm] \lambda_i, \mu_j [/mm] findet mit

$ [mm] \lambda_1\cdot{}\vektor{1 \\ 2 \\ -1 \\ 0}+ \lambda_2\cdot{} \vektor{1 \\ 0 \\ -1 \\ 0}+\lambda_3\cdot{}\vektor{0 \\ -1 \\ 3 \\ 2}= \mu_1\cdot{}\vektor{0 \\ 1 \\ 0 \\ 1}+\mu_2\cdot{}\vektor{2 \\ 2 \\ -1 \\ 1} [/mm] $.

Daraus ergibt sich ein homogenses LGS mit den Variablen  [mm] \lambda_1, \lambda_2, \lambda_3, \mu_1, \mu_2. [/mm]

Dessen ZSF lautet:


> > > [mm]\pmat{ 1 & 0 & 0 & 0 & -1 \\ 0 & 1 & 0 & 0 & -1 \\ 0 & 0 & 3 & 0 & -1 \\ 0 & 0 & 0 & 3 & 1}[/mm]
>  
> >  

> > >  

> > > ich erhalte den freien Parameter [mm]\mu_2=a[/mm] und
> > > [mm]\mu_1=-\bruch{a}{3}[/mm]

Genau.

Daraus weißt Du, daß gerade die Elemente aus [mm] Lin(v_4,v_5) [/mm] im Schnitt liegen, die von der Machart

[mm] -\bruch{a}{3}\cdot{}\vektor{0 \\ 1 \\ 0 \\ 1}+a\cdot{}\vektor{2 \\ 2 \\ -1 \\ 1}=a\vektor{2\\\bruch{5}{3}\\-1\\\bruch{2}{3}} [/mm]

sind.
Also ist der Vektor [mm] \vektor{2\\\bruch{5}{3}\\-1\\\bruch{2}{3}} [/mm] eine Basis des Schnittes.
Wenn's Dir besser gefällt, kannst Du auch ein Vielfaches davon nehmen, etwa [mm] \vektor{6\\5\\-3\\2}. [/mm]
---

Aber ich glaube, daß ich verstehe, was Dir Sorgen macht, nämlich, daß hier die anderen 3 Variablen nicht beachtet werden.

Wir lösen das jetzt noch etwas "anders":

Du hattest

[mm] $\pmat{ 1 & 0 & 0 & 0 & -1 \\ 0 & 1 & 0 & 0 & -1 \\ 0 & 0 & 3 & 0 & -1 \\ 0 & 0 & 0 & 3 & 1}$ [/mm] --> [mm] $\pmat{ 1 & 0 & 0 & 0 & -1 \\ 0 & 1 & 0 & 0 & -1 \\ 0 & 0 & 1 & 0 & -1/3 \\ 0 & 0 & 0 & 1 & 1/3}$. [/mm]

Hieraus (Bestimmung des Kerns, ggf. an anderer Stelle nachlesen) bekommst Du, daß alle Vektoren  der Gestalt

[mm] \vektor{\lambda_1\\\lambda_2\\\lambda_3\\\mu_1\\\mu_2}=a*\vektor{1\\1\\1/3\\-1/3\\1} [/mm] Lösung des oben aufgestellten Gleichungssystems sind.

Dh. es sind die Vektoren der Bauart

$ [mm] a\cdot{}\vektor{1 \\ 2 \\ -1 \\ 0}+ a\cdot{} \vektor{1 \\ 0 \\ -1 \\ 0}+1/3a\cdot{}\vektor{0 \\ -1 \\ 3 \\ 2}=a\vektor{2\\5/3\\-1\\2/3} [/mm]

bzw.

[mm] -1/3a\cdot{}\vektor{0 \\ 1 \\ 0 \\ 1}+a\cdot{}\vektor{2 \\ 2 \\ -1 \\ 1} =a\vektor{2\\5/3\\-1\\2/3} [/mm]

im Schnitt,

also ist [mm] \vektor{2\\5/3\\-1\\2/3} [/mm] eine Basis des Schnittes.






>  >  >  
> > > [mm]V={\mu_1*v_4+\mu_2*v_5/ \mu_2=a , \mu_1=-\bruch{a}{3}, a\in \IR}[/mm]
> > > = [mm]{a*\vektor{2 \\ \bruch{5}{3} \\ -1 \\ \bruch{2}{3}}}= {a*\vektor{6 \\ 5 \\ -3 \\ 2}}[/mm]
>  
> >  

> > >  

> > > also ist [mm]\vektor{6 \\ 5 \\ -3 \\ 2}eine[/mm] Basis von  
> > > [mm]Lin(v_1,v_2,v_3)\cap Lin(v_4,v_5).[/mm]
>  
>
> Ich verstehe nicht wie man darauf kommt! das ist mir nicht
> klar. Könnt iht mir das nochmal erklären?
>  
>
> MfG
>  Mathegirl


Bezug
                                
Bezug
Basis eines Unterraums: Mitteilung
Status: (Mitteilung) Reaktion unnötig Status 
Datum: 11:44 Mi 21.03.2012
Autor: Mathegirl

Dann hat sich das Problem jetzt für mich auch gelöst. Ganz richtig, ich habe immer alle Vektoren betrachtet und versucht daraus eine Basis zu basteln.

Es ist eigentlich logisch, dass es sich nur um diese 2 vektoren handeln kann..naja..wer nachdenken kann ist klar im Vorteil ;-)

Danke fürs Erklären, nun ist alles klar!

MfG
Mathegirl

Bezug
Ansicht: [ geschachtelt ] | ^ Forum "Lineare Algebra - Matrizen"  | ^^ Alle Foren  | ^ Forenbaum  | Materialien


^ Seitenanfang ^
ev.vorhilfe.de
[ Startseite | Mitglieder | Impressum ]